5
$\begingroup$

I am getting stuck with figuring out how to use Tooltip for a ListPlot3D. Basically I have a list of values for the 3D plot (no table or function) and would like to have the Tooltip return the x, y and z coordinates for the point.

Here is the code for the Plot that I would like to have "tooltipped":

correlationMatrix = {{1., -0.312997, -0.23282, 
  0.21541, -0.492404, -0.429369, -0.529716, -0.139237, 
  0.545136, -0.329623, 0.0439952, -0.281572, 
  0.311432, -0.339743, -0.313293, -0.276101}, {-0.312997, 1., 
  0.124945, -0.40165, 0.346201, 0.202914, 
  0.422802, -0.330172, -0.255637, -0.312864, -0.669181, 
  0.148235, -0.26647, 0.171766, -0.529144, 0.0928761}, {-0.23282, 
  0.124945, 1., 0.117204, -0.284851, -0.0106801, 0.132487, 
  0.266604, -0.612488, 0.276884, 0.194778, -0.169994, 
  0.508427, -0.430038, -0.0587014, -0.646482}, {0.21541, -0.40165, 
  0.117204, 1., -0.139717, -0.264258, -0.391223, 0.311072, 0.222882, 
  0.219147, 0.702512, -0.454245, 0.342072, -0.22791, 0.473594, 
  0.0226572}, {-0.492404, 0.346201, -0.284851, -0.139717, 1., 
  0.382168, 0.351843, -0.454952, -0.0969759, -0.234836, -0.355201, 
  0.169766, -0.190264, 0.337011, 0.264028, 0.509604}, {-0.429369, 
  0.202914, -0.0106801, -0.264258, 0.382168, 
  1., -0.24091, -0.279989, -0.0826109, 0.370576, -0.102702, 
  0.229024, -0.366183, 0.265442, 0.0947463, 0.225124}, {-0.529716, 
  0.422802, 0.132487, -0.391223, 0.351843, -0.24091, 
  1., -0.178734, -0.520441, -0.252357, -0.396323, 0.191239, -0.104731,
   0.0866192, -0.0556171, -0.0804755}, {-0.139237, -0.330172, 
  0.266604, 0.311072, -0.454952, -0.279989, -0.178734, 1., -0.050751, 
  0.659989, 0.515793, 0.0467429, 0.0373106, 0.0889927, 0.257039, 
  0.0444238}, {0.545136, -0.255637, -0.612488, 
  0.222882, -0.0969759, -0.0826109, -0.520441, -0.050751, 
  1., -0.0912518, 0.304593, -0.29235, -0.398841, -0.093443, 0.272338, 
  0.225868}, {-0.329623, -0.312864, 0.276884, 0.219147, -0.234836, 
  0.370576, -0.252357, 0.659989, -0.0912518, 1., 0.539342, 
  0.113201, -0.0615683, 0.133768, 0.362695, 
  0.0785236}, {0.0439952, -0.669181, 0.194778, 
  0.702512, -0.355201, -0.102702, -0.396323, 0.515793, 0.304593, 
  0.539342, 1., -0.573185, 0.0920204, -0.456162, 
  0.678595, -0.221534}, {-0.281572, 0.148235, -0.169994, -0.454245, 
  0.169766, 0.229024, 0.191239, 0.0467429, -0.29235, 
  0.113201, -0.573185, 1., -0.20389, 0.891105, -0.119537, 
  0.520205}, {0.311432, -0.26647, 0.508427, 
  0.342072, -0.190264, -0.366183, -0.104731, 
  0.0373106, -0.398841, -0.0615683, 0.0920204, -0.20389, 
  1., -0.368594, -0.205337, -0.353868}, {-0.339743, 
  0.171766, -0.430038, -0.22791, 0.337011, 0.265442, 0.0866192, 
  0.0889927, -0.093443, 0.133768, -0.456162, 0.891105, -0.368594, 1., 
  0.0188257, 0.824432}, {-0.313293, -0.529144, -0.0587014, 0.473594, 
  0.264028, 0.0947463, -0.0556171, 0.257039, 0.272338, 0.362695, 
  0.678595, -0.119537, -0.205337, 0.0188257, 1., 
  0.187089}, {-0.276101, 0.0928761, -0.646482, 0.0226572, 0.509604, 
  0.225124, -0.0804755, 0.0444238, 0.225868, 0.0785236, -0.221534, 
  0.520205, -0.353868, 0.824432, 0.187089, 1.}};

ListPlot3D[correlationMatrix, AxesStyle -> Thickness[0.01], 
 AxesLabel -> {"X", "Y", "Corr(X,Y)"}, 
 AxesEdge -> {{-1, -1}, {-1, -1}, {-1, -1}}, 
 ColorFunction -> "BrightBands"]
$\endgroup$
2
  • 1
    $\begingroup$ How can you make a ListPlot3D without a table? Could you provide a simple code example of what you mean to do? $\endgroup$
    – Jens
    Jul 2, 2012 at 18:53
  • $\begingroup$ Good suggestion. I added the data and the code. $\endgroup$
    – FredrikD
    Jul 2, 2012 at 19:09

1 Answer 1

7
$\begingroup$
Show[
  ListPlot3D[correlationMatrix,
    AxesStyle -> Thickness[0.01], 
    AxesLabel -> {"X", "Y", "Corr(X,Y)"}, 
    AxesEdge -> {{-1, -1}, {-1, -1}, {-1, -1}}, 
    ColorFunction -> "BrightBands"], 
  ListPointPlot3D @ Tooltip @ Flatten[
    MapIndexed[Flatten@{#2, #1} &, correlationMatrix, {2}], 1]
]

enter image description here

$\endgroup$
1
  • $\begingroup$ Clever solution! $\endgroup$
    – FredrikD
    Jul 2, 2012 at 20:30

Your Answer

By clicking “Post Your Answer”, you agree to our terms of service and acknowledge you have read our privacy policy.

Not the answer you're looking for? Browse other questions tagged or ask your own question.